Вы находитесь на странице: 1из 14

MOCK XAT

Answers and Explanations


1 B 2 C 3 C 4 E 5 D 6 D 7 B 8 D 9 C 10 A
11 E 12 D 13 B 14 D 15 A 16 B 17 C 18 C 19 B 20 D
21 B 22 B 23 D 24 B 25 C 26 D 27 A 28 C 29 B 30 A
31 D 32 D 33 C 34 E 35 C 36 B 37 A 38 D 39 A 40 B
41 D 42 E 43 D 44 A 45 E 46 D 47 E 48 B 49 C 50 A
51 C 52 A 53 B 54 D 55 D 56 E 57 C 58 B 59 C 60 E
61 B 62 A 63 C 64 D 65 E 66 E 67 C 68 A 69 A 70 B
71 C 72 A 73 D 74 C 75 D 76 D 77 B 78 C 79 D 80 E
81 C 82 D 83 D 84 E 85 E 86 C 87 A 88 C 89 C 90 B
91 A 92 B 93 C 94 C 95 C 96 A 97 A 98 B 99 D 100 C
101 E

MY PERFORMANCE
Time
Total Total Correct Incorrect Net
Taken
Questions Attempts Attempts Attempts Score
(Min)
Verbal Ability and Logical
34
Reasoning
Analytical Reasoning and
24
Decision Making
Data Interpretation and
43
Quantitative Ability

TOTAL 101 120

Discuss this test online at PREPZONE


http://www.careerlauncher.co.in/prepzone

MCT-0004/11
MOCK XAT Page 1
1. B Statement (B) directly contradicts the Theory of Constraints 7. B Whether fits into the first blank. The sentence is not conditional.
which states that businesses should scale their entire It can be rephrased as Regardless of whether you have a
production to the process within the system with the lowest doubt or not &c. For the second blank the correct choice is
capacity; the bottlenecks. This will make the system appear out. Ferret out, in the context of the sentence, means to
less efficient, because areas in the factory may stay idle if discover pertinent research papers after careful searching.
they have a much higher capacity than bottleneck areas. At your own risk is the correct expression.
However, implementing this approach will directly add to the
bottom line. So the focus is clearly not on ensuring every 8. D The odd pair is circumspect /indiscreet. The rest of the pairs
machine runs at maximum efficiency. The rest of the options consist of some disciplinary/punitive action and its antonym.
are aligned with TOC. Operational expenses and inventory
are said to stay at a desirably low figure. 9. C (A) is incorrect as it is mentioned that wonders of Nature are
“passage towards the wonders of art” but nothing has been
2. C From the passage it’s clear that the Theory of Constraints said about the role of Nature or its wonders in the conception
focuses on processes that are bottlenecks in the system. of any piece of art. Similarly in the same sentence, it’s
This implies that there are clearly defined processes that can intelligence that is nearest the wonders of Nature and not art.
be identified and tweaked to achieve certain objectives. Thus, (B) is also incorrect. (D) can be eliminated as nothing is
Statement 1 is not in line with TOC which stresses the need mentioned about intelligence being a rarity in Art. Further, the
for keeping operational expenses under check by not chasing lines, “For it is not yet known in what cases and how far
efficiencies blindly. The passage mentions dependent events effects attributed to superstition do participate of natural
and statistical fluctuations. So TOC clearly does not depend causes...” makes (E) an incorrect choice.
on precise predictions/forecasting. The passage also states Refer to the lines, “...howsoever the practice of such things
Keeping large amounts of inventory will further subtract from is to be condemned..., but for the further disclosing of Nature.”,
the bottom line, because warehouse space is costly. So and the last sentence, “they are either true or not natural; and
statement (E) is eliminated. Statement (B) is incorrect. While therefore, impertinent for the story of Nature.” This enables
the emphasis of TOC is not on maximizing the efficiency of us to infer (C).
each worker/machine, this does not imply in any way that low
efficiency is essential for TOC to work. 10. A Options (B) and (C) can be eliminated as the author is
concerned not just with the negatives or positives in the work
3. C Statement (C) follows directly from the passage. The passage of art but both; specifically in the context of the use of
states that Goldratt’s business model is based on two supernatural in it. This, in fact, makes (A) the right answer
principles. The first principle defines three ways to measure choice. (D) is eliminated as the author is not exclusively talking
whether or not businesses are achieving the goal of making of the role of supernatural in Nature. Moreover, that is not his/
money. So statement (A) is eliminated. Additionally, TOC her main concern. (E) is only partially correct and not the main
involves scaling the entire production to the process with the concern of the author.
lowest capacity, not necessarily eliminating such processes.
Since TOC takes into account dependent events as well as 11. E Statement (E) is aligned with the passage given above. The
statistical fluctuations, statements (B) and (D) are eliminated. author challenges the view that theories exist only because
Also, it is not implied in the passage that TOC shuns production the rule is violated at every turn.
schedules. Statement (E) contradicts the basic tenet of TOC
which focuses on the capacity of the entire system. 12. D Statement (D) gives a clear rationale for the rule that a theory
that contradicts experience must be excluded from science
4. E Statement (E) gives the reason why TOC can involve limiting and replaced by a better theory.
production to traditionally inefficient rates. The throughput of
the entire system can only be increased if the rate of throughput 13. B The sentence claims that apparent instances of spontaneity,
at the area of lowest capacity is increased. Statement (D) is creativity and altruism cannot be explained away by
correct, but it does not make it clear why limiting production materialist, determinist and self-interest theories which
to traditionally inefficient rates is justifiable. Statements (B) nevertheless try to do so. These theories translate instances
and (C) are incorrect. TOC does take inventory and operational of spontaneity, creativity etc to something consistent with
expenses into account. Statement (A) directly contradicts materialist preconceptions and in the process reduce the
TOC which focuses on the entire system, unlike more traditional unfathomably complex to the readily comprehensible. The
models some of which operate on a microlevel, accounting first blank needs a word that requires less involvement on the
for specific areas of the system instead of focusing on the theorists’ part than even explaining. The correct option is
entire system. acknowledging. The second blank needs a word that hints
that these experiences are reductively translated, that they
5. D The sentence challenges qualia as a venerated term that has go unacknowledged. The correct expression is whisked away
not been adequately analyzed. So, supposedly fits into the which in the context means to be removed. Since the sentence
first blank. The notion of qualia is said to have become a says that the unfathomably complex (what our senses
criterion or standard by which new research is judged. So perceive) is reduced to the readily comprehensible (through
the sentence must talk about challenging this orthodoxy-this materialist preconceptions), the only word that can fit into the
importance given to qualia. Option (B) is eliminated because third blank is consistent.
vanishingly does not go with irreducible. Totemic is the
correct choice for describing qualia in the context of the 14. D Statements I & III can be concluded from the passage. According
sentence; qualia have become venerated symbols and have to the passage, the worldview of a person is the result of
gone unchallenged for many years. arranging perceptions into images by imagination. This means
that to understand a person’s worldview, his/her perceptions
6. D The correct option is (D). To zero in means to close in. lesser need to be taken into account. It also follows that imagination
should be followed by than here. To check on something shapes perceptions into images. So imagination is required to
means to look into the legitimacy or condition of someone or develop a worldview. Statements II and IV are assertions not
something. supported by the passage.

Page 2 MOCK XAT


15. A Statement (A) provides evidence that goes against the popular 23. D Haul someone over the coals means to reprimand someone.
claim that children need to be in close contact with their parents So (D) is the odd one out. The idiom and the phrase next to it
during the first three years of their lives. Statement (E) is a do not match. Half a bubble off plumb means to be giddy or
general statement that does not specify that children need to crazy. A square deal is a fair and honest transaction.
develop close connections with their parents; just that they Something is in the bag means something has been or is as
need to develop close connections. good as achieved. Run a make on means to perform an
identity check on someone.
16. B Only (B) provides proof for the importance of close parental
connections for a child thereby weakening the sociologists’ 24. B IV, III, I, II is the correct sequence. The passage distinguishes
argument. between the ethic of justice rights and the ethic of care. IV
introduces the topic. Sentence II is a conclusion that the ethic
17. C The statement in (C) is not implied in the passage. According of care stresses connection with others both in what it says
to the passage, the implications of a singular use of a word about the normative basis of morality and in what it says
can only be assessed by looking at the text in entirety. This about the ways in which moral goodness shows itself within
does not imply that the secondary meanings of words cannot a morally good life and that the Kantian views lay stress on
indicate the subject matter of the text. being guided by moral principles or judgments within the
moral life. These two points are covered in III and I respectively.
18. C Patrons spend more money buying products when classical
music rather than recent hits are played in the background. 25. C The passage starts out by talking about Iveta Radicova and
It’s irrelevant if consumers have different tastes in classical how she thinks that the SAS party has sabotaged a bill.
music. What is being compared is classical music and recent Sentence II follows sentence IV by elaborating on Radicova’s
hits. So statement (E) does not point to any oversight. Similarly response. Sentence III shifts the passage to a discussion on
even if customers visit the store because of the range of whether Slovakia’s disobedience will become contagious,
products available there, this does not affect the owner’s emboldening other eurozone parliaments to follow its lead.
strategy (playing classical music in the background and Sentence V moves from the genera statement in sentence III
stocking up on items in every price range). Statement (A) to a particular economist who endorses the view described
similarly does not add to the argument in any way. Statement in sentence III. Sentence I provides a counterargument to this
(B) does not identify an oversight in strategy since nothing is stance. Sentence VI continues to elaborate on her view of
said about what proportion of customers is less responsive the no vote in Slovakia. According to her, Slovakia’s example
to classical tracks. Moreover, it does not state that such patrons will act as a deterrent for other countries.
buy less or are unresponsive when classical tracks are The correct sequence is IV, II, III, V, I, VI.
playing. So statement (B) is also irrelevant. Statement (C)
points to a flaw in the owner’s strategy. The number of items 26. D The argument rests on and cites statistics based on the
sold in lower price ranges is going to go down if he plays assumption that the presidential candidate will endorse the
classical tracks inside the shop. So he should have stocked views of the majority of the party members.
up on items in higher price ranges rather than indiscriminately
stocking up on items in all price ranges. 27. A The answer comes out clearly from the first line –“ Steven
Sverdlik’s Motive and Rightness is an impressive and wide-
19. B The correct option is (B). The rebels, violently reacting to the ranging treatment of an important but relatively under-explored
truisms (a trite or banal remark or statement, especially one question”. It is clear that the author is reviewing Steven
expressed as if it were original or significant) of the politician Sverdlik’s Motive and Rightness . what he is reviewing
can only be described as truculent. The correct idiomatic becomes clear from this line “Consideration of this explanatory
expression is tug at the heartstrings (to cause strong feelings question makes up the bulk of the book (Chs. 3-7), with the first
of affection). two chapters introducing terms and examples, and the last
two chapters answering additional questions that arise along
20. D The correct expression is trip up which means to cause to the way.
make a slip or error. The witness makes a contradictory
statement and this makes him trot out the truth. 28. C The answer to this question can be arrived at from a reading
of the last paragraph , where the author acknowledges the
21. B The correct sequence is presence of certain conditions/ situations where the motive
, → ,→ . → ,→ ,→ . might affect the deontic status of the actions. He, however,
The greater the reliance on pre-capitalist economic relations, does not say they always do( read the last sentence of the
then, the more successful capitalism is in raising the rate of passage). The meaning of the word ‘deontic’ can be gathered
surplus value. Let us note that if the process of relying on from these lines from the 1st para –“Can the motive of an
household production were carried to its limit, the value of action affect its deontic status? (That is to say, does an action’s
variable capital would disappear, and with it wage labor. motive ever determine whether that action is right or wrong?)”

then needs to be in parenthetical commas in this case, so that 29. B Only statement (B) is logically consistent with the contents of
the meaning of the sentence is complete even without then. the paragraph. It states that patients report greater
(The greater the reliance on pre-capitalist economic relations, improvements than what has actually happened; that their
the more successful capitalism is in raising the rate of surplus reports are ‘upwardly biased’ and the effect of the treatment
value). is less than what is reported. Attention should be paid to the
word ‘tangible.’ Here it has the import of ‘real’ or ‘concrete’-
22. B From the sentence it's clear that the parents were against the not the reported improvement due to the Hawthorne effect,
person's taking up the project, but on realizing that their mind but the actual, tangible improvement in pain.
was made up, relented. Only dissuade and prevent can fit
into the first blank. convince, reason and persuade cannot be
used with from. hamper is a transitive verb and cannot be
used in the second blank.

MOCK XAT Page 3


30. A Going by Blurb’s views, he is most likely to object to statement 36. B The shortest possible route will be F, C, D, E, B and A. The
(iv). According to him all genuine questions must have one table given below shows the total time taken by the delivery
true answer and one only, all the rest being necessarily errors. boy to reach one’s place from the Delivery Unit.
This is incompatible with the view that conflicting ideas can
be close to reality. Blurb does not elaborate on whether Time Taken
perceptual experience is closer to reality than dream worlds Person
or vice versa. So statement (i) is incorrect. Statements (ii), (iii) (in minutes)
and (v) do not contradict his views here. F 2
C 8
31. D The correct option is (D). According to Stake, to conceptualize
reality better, chaotic perceptual experience needs to be D 18
transformed by intelligence and can be used to create dream E 32
worlds. Statement (A) is incorrect because Stake does not B 38
imply that reality should conform to our desires. He does
A 46
speculate that some of the dream worlds we create might be
closer to our desires. Statement (B) does not follow. While
Stake speculates that other possible worlds might be The time taken to reach the Delivery Unit from A’s place is
discovered which are more rational and intelligible than the 2 + 6 = 8 minutes. Therefore, the total time taken by the delivery
one we find ourselves in, he does not make a definite boy is 46 + 8 = 54 minutes.
connection between rationality and reality. Statement (E) is 37. A The shortest possible route will be A, D, F, C, B and E. The
incorrect. From Stake’s statements it appears that he believes table given below shows the total time taken by the delivery
the universe we inhabit is not as close to our soul as it should
be-“ more akin to the soul than this strange universe which boy to reach one’s place from the Delivery Unit.
man has hitherto always looked upon with increasing
astonishment.” Time Taken
Person
(in minutes)
32. D Review (D) objectively appraises the new book, examines it A 6
without referring to the conventions of the genre. Review 1
D 10
talks about most other self-help books. It seems that the
reviewer is operating with some fixed notions about what a F 18
self-help book should be like and the review seems to be C 24
more of a sales pitch.
B 34
33. C Review (C) rests on an assumption that the book’s readership E 40
will not be able to identify with what the reviewer calls
“managerial jargon.” This does not consider the possibility The time taken to reach the delivery unit from E’s place is
that the readers might be familiar with managerial jargon. 2 + 8 = 10 minutes. Therefore, the total time taken by the
delivery boy is 40 + 10 = 50 minutes.
34. E Review (E) eulogizes Dale Carnegie and hardly talks about 38. D As Harsil and Giri form a couple, the only possible arrangement
the merits of the new book. is:

For questions 35 to 37: Left to right - order of arrival from first to last
Let Arjun, Boman, Crystal, Dhiraj, Ekansh and Fiza be represented by Husband Harsil/Giri Bindu Akshay Chandar
A, B, C, D, E and F respectively. As the speed at which the delivery Giri/Harsil Divya Fatima Ekta
Wife
boy travels is 30 km/hr, the time taken to travel a distance of d km
Therefore, we can conclude that only statement (i) is true and
d
= × 60 = 2d minutes. statements (ii), (iii) and (iv) are false.
30
39. A As Akshay came before Bindu, Fatima and Ekta must belong
35. C The shortest possible route will be F, C, D, A, B and E. The to the third and fourth couple to arrive, in no particular order.
table given below shows the total time taken by the delivery Chandar must be married to Ekta as it is given that he arrived
boy to reach one’s place from the Delivery Unit. after Fatima. From statement (iii), Giri must be married to Akshay
while Harsil must be married to Fatima. The above conclusions
can be shown as given below.
Time Taken
Person
(in minutes) Left to right - order of arrival from first to last
F 2 Husband Akshay Bindu Harsil Chandar
C 8 Wife Giri Divya Fatima Ekta
D 18
A 22 For questions 40 to 42:
Let Mumbhai, Punhe, Dehli and Kohlkata be represented by M, P, D and
B 30
K respectively. From statement (iii), it can be concluded that Air-2
E 36 operates on M-P, M-D, P-K and D-K. From statement (i), Air-1 must be
operating on five direct routes and hence it definitely operates on P-D,
Therefore, B and E will get their orders for free. P-K and D-K. From statement (iv), the number of routes on which
Air-4 and Air-5 are operating must be two each. Thus, it can be
concluded that Air-4 operates on M-D and P-K whereas Air-5 operates
on M-P and D-K. As at least one airline operates on each route, Air-1
must be operating on M-K.

Page 4 MOCK XAT


The conclusions made thus far can be shown as given below. From statement (iv), break-up of Biloza’s score must be 9 + 1 + 1. So
sum of the scores of Biloza and Preeti becomes the same i.e. 10 in all
M -P M -D M -K P-D P-K D-K three rounds.
From statement (v), break-up of Rajat’s score must be 9 + 5 + 1.
Air-1 √ √ √ √
Air-2 √ √ X X √ √ Break-up of score of all the candidates is shown in the table given
Air-3 X below.
Air-4 X √ X X √ X
Nam e Br e ak -up of s cor e
Air-5 √ X X X X √
A purv 5+5+5
Ekansh 5+5+9
40. B Punhe-Dehli
Biloza 9+1+1
41. D As Air-3 doesn’t operate on D-K, it doesn’t operate on P-K as Rajat 9+5+1
well. Hence, it must be operating on M-P, M-D and P-D. The Neeru 5+5+1
final table can be shown as given below. Preeti 9+9+1
Sharad 9+9–3
M-P M-D M-K P-D P-K D-K
Hasan 9+5–3
Air-1 √ X √ √ √ √
Air-2 √ √ X X √ √ 43. D Hasan and Sharad
Air-3 √ √ X √ X X
Air-4 X √ X X √ X 44. A Apurv and Neeru
Air-5 √ X X X X √ 45. E Preeti and Sharad

For questions 46 to 48:


Therefore, the number of airlines operating on Punhe-Dehli
Let India, Pakistan, Australia, Kenya and England be represented by I,
route is different from the rest three routes.
P, A, K and E respectively.
There were ten pairs of teams and hence twenty matches must have
42. E As the number of airlines operating on M-D and P-D is the
been played.
same, the number can be 2 only. The final table can be shown
From statement (ii), both I and E must have played at least four matches
as given below.
at home. From statements (iii) and (i), the two matches between K and
I were played in different countries i.e. one in K and one in I. Combining
M-P M-D M-K P-D P-K D-K this conclusion with statement (v), I must have played both the matches
Air-1 √ X √ √ √ √ against P and E at home. Therefore, the total number of matches
played in I was 6.
Air-2 √ √ X X √ √
Air-3 X X X √ √ √ 46. D As exactly 3 matches were played in K, from statement (ii), E
Air-4 X √ X X √ X must have played both the matches against P and A at home.
As there were five pairs of teams between whom the two
Air-5 √ X X X X √
matches were played in two different countries, the two
Therefore, the number of airlines operating on P-K and D-K is matches between P and A must have been played in different
more than 3. countries i.e. one in P and one in A. The conclusions made can
be tabulated as shown below.
For questions 43 to 45:
The different possible points scored in any round could be 9 (all three Teams Venue I / II Venue II / I
correct), 5 (two correct, one wrong), 1 (one correct, two wrong) and
–3 (all three wrong). I–P I I
I–A I A
The table given below shows the possible break-up of the candidates’ I–K I K
scores in the three rounds (in no particular order) at the end of the
I–E I I
selection process. The number of questions answered wrongly by
the candidates in the three rounds together is mentioned in brackets P–A P A
along with their total scores. P–K P P
P–E E E
19 15 11
A–K K A
(Two wrong) (Three wrong) (Four wrong)
A–E E E
Case I 9+9+1 9+9–3 9+5–3
K–E K E
Case II 5+5+9 5+5+5 5+5+1
Case III - 9+5+1 9+1+1
Therefore, 3 matches were played in Pakistan.
From statement (ii), break-up of Apurv’s score must be 5 + 5 + 5 and
break-up of Preeti’s score must be 9 + 9 + 1. Therefore, break-up of
Ekansh’s score must be 5 + 5 + 9.
From statement (iii), break-up of Neeru’s score must be 5 + 5 + 1. So
Neeru and Apurv must definitely have scored equal points in two of
the three rounds.

MOCK XAT Page 5


For questions 47 and 48: 52. A Option (A) should have been the management’s approach.
As exactly four matches were played in E, E must have played both The company is said to have been plagued by labour problems
the matches against K in E. Either A or P must have played both its throughout its history. Rather than addressing this issue, the
matches against E at home. However, this country cannot be P as it management has gone for aggressive restructuring which
will violate statement (ii). can only lead to lowered employee morale. Option (B) gives a
As the number of matches played in five countries are five consecutive general guideline, but no effective cues on implementation.
integers, the only possibility is 2, 3, 4, 5 and 6. Therefore, K must have Option (C) is worse as it relieves the management of their
played two matches at home and P must have played three matches duties and does not include the employees in decision making.
at home. The conclusions made can be tabulated as shown below. Option (D) is inappropriate as it relies on an external consultant
thereby further distancing the management from the
employees. Option (E) is a long-term process that should only
Teams Venue I / II Venue II / I be entered into if the organization has assessed this as a
I–P I I need. From the information in the passage, it can be assumed
I–A I A that this is not so.
I–K I K
53. B The immediate course of action would be to address each
I–E I I issue and ensure that future problems do not crop up. Option
P–A P A (B) fits in here. Option (A) is not correct. Each complaint
P–K P P needs to be investigated, not acted upon without inquiry. Option
(C) is ineffective as an immediate course of action . Option (D)
P–E E E is an incorrect decision as it is clear from the caselet that the
A–K A K VP-HR has already got an initial understanding of the situation.
A–E A A What is immediately required is intervention by the top
management. Option (E) is too vague an approach to be
K–E E E
effective.

47. E 4 54. D Since it is clearly stated that employees often work a 12-hour
shift, it can be inferred that the training programme must be
48. B 5 easily accessible and not interfere with their work schedule.
Option (D) is the answer and since HR will follow up it will not
49. C In a conflict situation, it is necessary to first work on bringing remain an ad-hoc training solution. Option (A) is incorrect as
in calm. Then action can be taken and a solution worked out. there is no information on industry norms for work hours, or
Option (C) works on bringing in calm through negotiations and whether the employees already work weekends by rotation
then an agreement is a step towards a solution. Option (A) is and so on. Option (B) may serve to demotivate the employees
incorrect as it is a means of avoiding a situation and not facing and would be detrimental to the training objectives. Option (C)
it. Option (B) is incorrect as it a ready acceptance of defeat is incorrect as it is vague – would the employees be expected
and will reflect poor judgment. Employees were asked to to take time off from work or work extra hours. Also, there is
leave for a reason and it would be negative on the no information on the validity of the frequency of training.
management’s part to readily accept a mistake on this unless Option (E) is clearly incorrect as it aims to take non-supervisory
there were sound basis for it. Option (D) cannot be a solution staff on an outbound training. Given that these employees
as it depends on the performance guarantee of workers. work 12-hour shifts it is clear that there is a heavy workload.
Option (E) is also incorrect as it would be futile to convince The case also states that there is an increasing amount of
workers of the validity of a decision they are agitating against. turnover and one can infer that there would be cases of short
staff experienced at a few sites atleast.
50. A The first paragraph of the case states that Mr. Preston is
known as the ‘turnaround specialist’. For four years he was 55. D The first step for any major change should be to identify if
also able to bring in a positive influence on XYZ’s performance. change is required and if so then how much of it is necessary
Option (A) is the answer. Option (B) cannot be inferred even and pragmatic. (4) should be the first step. Next, would be the
though there is information on the number of managers and identification of how much change the organization can make/
juniors who were asked to leave the organization after he absorb i.e. (2). The next course of action would be to draw
joined it. Option (C) also can’t be inferred as it is not certain on up an action plan – (3). (5) is a detail under (3) and therefore
what basis Mr. Preston appointed the expatriates in the top appears after it. (1) will be the last as it can only be done once
management positions. Option (D) is incorrect as there is no a road map is drawn and timelines are set and processes are
information on Mr. Preston working as an unbiased leader or identified. The correct order is 42351. Hence the answer is
a charismatic one. Option (E) is incorrect as it is not supported option (D).
by the information in the caselet.
56. E Smooth transition will occur when employees agree with the
51. C It is clear from the information in the case that human resources proposed changes and understand the reason for them.
policies needed to be changed – Preston and his team of new However, presently there are negative rumours about the
managers made changes and even shifted a few departments change that the management will propose. It would thus be
and functions to XYZnagar. Regular strikes and lockouts are necessary for the management to correctly communicate the
also indicative of a weak human resources and labour relations proposed change and clearly inform the employees about
policy, or in case a foolproof policy is in place, some gaps in what they can expect in the future. Certainly news from the
its implementation. The answer is option (C). Options (D) and management will be much better (or much greater) than news
(E) are incorrect as they are specific instances that happened received from the media. Option (E) is the answer. Option (D)
later. They are not the inherent cause of XYZ’s problems. will have the opposite effect than what is proposed and will
serve to worsen the situation. Option (A) is incorrect as the
number of employees increasing after the change is in itself
not enough to make employees accept change, or put up with

Page 6 MOCK XAT


any dismissals that might happen (consider a situation where For ‘n’ letters and ‘n’ envelopes, numbers of possibilities such
x old employees are fired and (x + 1) new, better-qualified that no letter is placed in the correct envelope is given as:
employees are recruited.) Also it needs to be considered
 1 1 1 1
whether increasing employee strength is a viable option. n! 1 − + − + ...... + ( −1)n . 
Option (B) is not likely to ensure a smooth transition as it will  1! 2! 3! n! 
confirm the employee’s misgivings regarding downsizing. So using the formula, we get
 1 1 1 1
57. C Since there is a conflict between both departments with each 4! 1 − + − +  = 12 − 4 + 1 = 9
one thinking the other is doing its job, it is clear that employees  1! 2! 3! 4! 
need to be made aware of how the other department works
and how integral it is to the organization. Administrative Hence, number of favourable cases is C4 × 9
501

procedures can also minimize conflict by ensuring fair play


and by making the functioning of both departments more C4 × 9
501
Therefore, required probability is .
systematic and in sync which brings us to points 4 and 5. 501!
Option (C) is thus the answer.
60. E A squadron from Dagra air base covered more percentage
58. B Drinking in office is a serious offence. However, the role of area in Defence Mode as compared to a squadron from Jambala
the person involved heightens the scope of the problem. air base. The percentage of fighter jets lost per squadron per
Removing a Director from his position immediately will not be flight was also lower for Dagra in Defence Mode. Hence, it
an easy task. Therefore it is best to ask him to take a leave of would be wiser to use Dagra in Defence Mode and Jambala in
absence while investigations are done and a decision is Attack Mode, to minimise the damage during combat.
reached. Also the incident would have negated Chandra’s In every 12 hours, Dagra air base would need 8 squadrons
credibility as Director. It would be best for him to remove (Since 8 × 12.5% Territory = 100% Territory) i.e. 8 × 12 = 96
himself from the scene. Option (B) is the answer. Option (A) fighter jets in Defence Mode. 8.33% of these would be lost
is incorrect as it is hasty and may not follow correct protocol per flight during combat i.e. 8 fighter jets would be lost in
by leaving Chandra out. Even if eventually Chandra is asked every 12 hours. So in 24 hours, 8 × 2 = 16 fighter jets would
to leave, it should be done after evidence is gathered and be lost.
procedures are taken into account. Option (C) is incorrect as Similarly, in every 8 hours, Jambala air base would need 12
it depends on hope and the situation demands immediate action. squadrons (Since 12 × 8.33% Territory = 100% Territory) i.e.
Option (D) is incorrect as this is not deal with Chandra and 12 × 12 = 144 fighter jets in Attack Mode. 33.33% of these
avoids immediate action. Option (E) is incorrect as it again would be lost per flight during combat i.e. 48 fighter jets would
avoids the correct and immediate action – it bases itself on be lost in every 8 hours. So in 24 hours, 48 × 3 = 144 fighter
trusting a change in the behavior of someone who has no jets would be lost.
control over his situation. Hence, the minimum number of fighter jets lost by the two air
bases in 24 hours would be 16 + 144 = 160.
59. C Total number of ways of placing letters in the envelopes is
61. B If the number of squadrons being used from Jambala air base
501! was 5, then the number of squadrons to be used from Dagra
air base would be 4. (Since 5 × 10% Territory + 4 × 12.5%
Number of ways of selecting 4 letters = 501C4
Territory = 100% Territory)
Number of ways of placing four letters in 4 different envelopes The number of fighter jets lost = 3 × 5 × 4 + 1 × 4 × 4 = 76.
is 4! or 24.
Number of ways of placing each of ‘n’ letters out of the 4 in 62. A In the first 12 hours, Jambala air base would need to use 10
correct envelope is: squadrons (Since 10 × 10% Territory = 100% Territory) in
Defence Mode. It would lose 25% of its fighter jets
n Number of ways i.e. 2.5 squadrons in this period. For every successive 12
hour periods, it will continue to add 2.5 squadrons to maintain
4 1
the strength of 10 squadrons in Defence Mode. The maximum
3 0 number of hours for which the entire territory of Ladakabad
2 4 C × 1 or 6
2 20
could be defended = 12 + × 12 = 108 hours.
1 4 C × 2 or 8 2.5
1
0 x
63. C Let the required number be ‘n’ and the sum of the digits be S.
Therefore, 275 < n – S < 300
x = 24 – 1 – 6 – 8 = 9 ways
Thus, we can conclude that ‘n’ must be a three-digit number
501C ×9
4 and 1 ≤ S ≤ 27.
Required probability =
501! Let the number be 100x + 10y + z. It we subtract
x + y + z from this, we get 99x + 9y, which is clearly a multiple
Alternate solution: of ‘9’.
Total number of ways of placing letters in the envelopes is
501! The possibilities are 279, 288 and 297.
Number of ways of selecting 4 letters which were placed in
501 If 99x + 9y = 279, we get
the wrong envelope is C4 .
11x + y = 31

MOCK XAT Page 7


The only possibility for ‘x’ and ‘y’ is 2 and 9 respectively. ‘z’ 67. C Let the total number of men be 100x and the total number of
can take any value from ‘0’ to ‘9’. So there are 10 such numbers. women be 100y.
Given, 5x = 2 × 33y and 100y = 1,000.
If 99x + 9y = 288, we get Therefore, 100x = 13,200.
11x + y = 32 So the number of men who read at most two newspapers
= 95x = 12,540.
As ‘y’ must be less than or equal to 9, No solution is possible.
68. A The percentage of men who read only ET would become
If 99x + 9y = 297, we get 25 + 9 + 12 = 46% and those who read only HT would become
11x + y = 33 20 + 7 + 8 = 35%. So the percentage of men reading ET would
The only possibility for ‘x’ and ‘y’ is 3 and 0 respectively. become 100 – 35 = 65%.
‘z’ can take any value from ‘0’ to ‘9’. The percentage of women who read only ET would become
So again there are 10 solutions. 15 + 5 + 6 = 26% and those who read only HT would become
40 + 13 + 6 = 59%. So the percentage of women reading ET
So, total such numbers = 10 + 10 = 20. would become 100 – 59 = 41%.
64. D Insertion of a new digit will change the base system from So the number of copies of ET in circulation
10 to 11 and hence the value of δ will be 5. In the new base
65 41
system, value of all the digits greater than or equal to 5 will = × 20,000 + × 15,000 = 19,150
increase by 1, i.e. value of 5 will be 6, value of 6 will be 7 and 100 100
so on. 69. A The total number of men who read HT in 2010
Therefore, (9δ5)11 = 10 × 112 + 5 × 111 + 6 × 110 = 1271 = 4,600 × 1.3 = 5,980
Therefore, distance traveled = 1271 km. The total number of men who read ET in 2010
65. E To break level number 1, there is only one possible numeric = 5,300 × 1.2 = 6,360
sequence: 1 Increase in the number of readers of:
To break level number 2, there are two possible numeric HT = 1,380 and ET = 1,060.
sequences: 1 + 1 and 2 To minimize the number of men who did not read any
To break level number 3, there are three possible numeric newspaper in 2010, we need to maximize the number of men
sequences: 1 + 1 + 1, 2 + 1 and 1+2 who read only HT and only ET in 2010.
To break level number 4, there are five possible numeric
sequences: 1 + 1 + 1 + 1, 2 + 2, 2 + 1 + 1, 1 + 2 + 1 and So the number of men who did not read any newspaper in
1+1+2 2010 is at least 2,500 – (1,060 + 1,380) i.e. 60.

It can be generalized that if there are Cn+1 sequences for


(n+1)th level, then these sequences can be divided in two 1 1 1
70. B S= + +
categories: ( xy + 2z ) ( yz + 2x ) ( xz + 2y )
(i) If the first term of a sequence is 1, then rest of the terms
The cyclicity must be exploited and the denominators in
can be entered in cn ways.
each term must be made equal such that the three different
(ii) If the first term of a sequence is 2, then rest of the terms
terms can be added easily.
can be entered in cn-1 ways.
Therefore, cn+1 = cn + cn-1
We can write xy + 2z as xy + (x + y + z) z = (z + x)
Therefore, there are 8 i.e. (5+3) sequences for level number
(z + y). Similarly the other denominators can be factorized. So
5.
now, to make all of them equal we multiply the Numerator and
C6 = 13, C7 = 21, C8 = 34 and C9 = 55.
Denominator by the “missing” term to complete the cyclicity.
Maximum number of attempts to break all the levels.
For example – the missing term for (z + x)(z + y) is (x + y)
= C1 + C2 +… + C8 + C9 = 142.
which completes the cyclicity.
And then all the three terms must be added to get
66. E Let the probability of Rubbagubba being alive on 24th of the
month be P(A) and being dead be P(D). 2(x + y + z)
S=
4 (x + y)(y + z)(z + x)
C3 31
Required probability P(D) = 1 – P(A) = 1 − 7
= .
C3 35 4
=
(x + y)(y + z)(z + x)
For questions 67 to 69: The data can be represented in the form of
4
Venn diagrams as shown below. =
(2 – z)(2 – x)(2 – y)
HT ET HT ET 4
=
(8 – 4x – 4z + 2zx – 4y + 2xy + 2zy – xyz)

40 % 5% 15 % 14 % 25 % 4
20 % =
10 % 5% {8 – 4(x + y + z) + 2(xy + zy + zx) – xyz}
13 % 5%
7% 9%
4
=
12 % {8 – 4(2) + 2(xy + zy + zx) – 1}
20 %
Now, (x + y + z)2 = x2 + y2 + z2 + 2(xy + yz + zx).
H in du From this we get (xy + yz + zx) = – 6
H in du
4 4
W om e n Men So, S = =– .
(–13) 13

Page 8 MOCK XAT


71. C Let O be the center of the circle and BD be a diameter . The sons who received only Company B shares received 184 and
4136 shares respectively.

72. A
B
72 cm 73. D

74. C
A
O
75. D Fixed monthly rental = Rs. 350
4800 × 1.25
Charges for Local calls = = Rs.60
100
D C Charges for STD calls
= 48 × 1.50 = Rs. 72
Call charges on Roaming
= 40 × 2.5 = Rs. 100
In cyclic quadrilateral, using Ptolemy’s theorem: SMS charges = Rs. 50
AC × BD = AB × DC + AD × BC K (i)

Using statement I: 113.5


Total bill = 632 × = Rs.717.32 .
Given that, BD = 90 cm. 100
∆ BAD is in semicircle, so ∠A = 90°
76. D Total billed amount before tax
AD2 = BD2 − AB2 = 902 − 722
0.8
⇒ AD = 54 cm = 275 + 50 × 2 + 80 × 6.75 + 2000 × + 60
100
As lengths of BC and CD are still not known, = Rs. 991
statement I alone is not sufficient to answer the question. After adding Service Tax, the total bill = Rs.1,124.785
Using statement II: 77. B
Given that, DC = 45 2 cm
No additional information is given. Charges Wodafone Cheeers Slice Airkel Ideas
Hence, statement II alone is also not sufficient.
Fixed 250 300 275 350 325
Using statements I and II together: Rental

BD = 90 cm, AD = 54 cm, DC = BC = 45 2 cm Local calls 48 54 44.80 60 54


Now using equation (i), length of AC can be calculated. STD calls 112.50 157.5 140 112.50 108
Hence, option C. ISD calls 687.50 840 708.75 712.50 660

For questions 72 to 74: Roaming 50 45 55 50 45


Let the total number of shares distributed by Mr. Moneywaala be ‘x’. Add-On 90 135 90 135 120
Let the total number of shares received by the son who got shares of services
both the companies be ‘y’.
Local SMSs 250 200 125 125 250
Therefore, the total number of shares received by the two sons who
4y Bill Amount
got only Company A shares = after S. tax 1636.80 1904.65 1582.405 1699.50 1718.20
15
Also, the total number of shares received by the two sons who got
only Company B shares = 4y By using Ideas instead of Cheeers, he would have saved Rs.186.45
4y 79y in that month and not Rs. 129.50 as mentioned in the statement.
⇒x=y+ + 4y =
15 15
78. C As Local calls exceed 600 minutes, we can easily conclude
Since each son received at least 144 shares, that the Local calls would be the cheapest for Slice.
4y 4y Local SMSs would be the cheapest for Slice and Airkel.
≥ 144×2 (As is total number of shares with the two sons who STD calls would be cheapest for Ideas but in that case Local
15 15
SMS charges would be very high. STD calls would be second
got only Company A shares.)
cheapest for Airkel and Wodafone, out of which Airkel is the
79y better choice, taking into consideration the much lower Local
⇒ y ≥ 1080 and x = ≥ 5688. SMS charges.
15
For a third connection, among Slice and Airkel, Slice should be
x preferred, taking into consideration the lower monthly rental.
Given, 144 ≥ ⇒ x ≤ 5760, therefore, the only possible value of ‘x’
40 It is required primarily for sending Local SMSs since a
is 5688 (as only 5688 is divisible by 79 in the range). connection each of Slice and Airkel are already there to take
The corresponding value of ‘y’ is 1080. care of Local and STD calls respectively.
The sons who received only Company A shares received 144 shares Hence, a new connection each of Slice and Airkel will prove
each. to be the best choice.

MOCK XAT Page 9


79. D The three-digit number EEE can be written as E × 3 × 37. 82. D India will win the series if it wins 4 or more matches.
⇒ AB × 37 × 2 = E × 3 × 37 ⇒ AB × 2 = E × 3 Case I: India wins 4 matches.
This is possible only when AB = 12 and E = 8. 4 3
5 4
Probability P1 = C4 ×   ×  
7
[The product of E and 3 has to be greater than 20 as AB is a
9 9
two-digit number and 1 ≤ E ≤ 9 ].
54 54
80. E C and D are the points of contact of the two circles with the = 35 × 64 × = 2240 ×
97 97
common tangent CD. Join C1C and C2D. From C2 drop a
perpendicular on C1C at point L. Case II: India wins 5 matches.
5 2
5 4 54
Probability P2 = C5 ×   ×   = 1680 × 7
7
9 9 9
C1 Case III: India wins 6 matches.
12
9 A 3 5 4
6
54
Probability P3 = C6 ×   × = 700 × 7
C2 7
L 9 9 9
3 3
Case IV: India wins 7 matches.
C B D E
7
5 54
Probability P4 =   = 125 × 7
Clearly, C2L = CD 9 9
Probability that India will win the series
But C2L = (C1C2 )2 − (C1L)2 = 152 − 92 = 12 cm
54
Also, CB = AB = BD (Since, length of tangents from an external = P1 + P2 + P3 + P4 = 4745 ×
point to a circle is equal) 97

CB + BD CD C2L
∴ AB = = = = 6 cm 83. D There are three possibilities.
2 2 2 Case I:

A’
81. C Let the number of students who took part only in Drama be
'3x', number of the students who took part only in Debate be
'y' and the number of students who took part only in Dance be 30°

'z'.
A

D e ba te N o ne =2 9

y B D C

We have, 60° < ∠DAC + ∠DA 'C < 120°


(As ∠DA 'C = 30° and 30°< ∠DAC < 90°)
y+1 y+1
10 Note: Line segment A’D is the bisector of ∠BA 'C, ∠BAC, and
side BC.
z x 3x
Case II:

D a nce D ra m a
A’
Therefore, y + 1 + y + 1 + y + 10 = 201
or y = 63 3 0°
4x + 3y + 2 + z + 10 = 320 –29
⇒ 4x + z = 90 …(i)
As the number of students who took part in Dance is 27 more
than the number of students who took in Drama, we have B D C
x + z + y + 1 + 10 = 3x + x + y + 1 + 10 + 27
⇒ z – 3x = 27 …(ii) We have, 30° < ∠DAC + ∠DA 'C < 60°
Solving equations (i) and (ii), we get
(As ∠DA 'C = 30° and 0°< ∠DAC < 30°)
x = 9 and z = 54
Therefore, number of the students who took part in Dance
= 137.

Page 10 MOCK XAT


Case III: For questions 85 to 88:
Let Net Expenditure for the month of January be Rs.100x.
A Profit in January = Rs.10x
Revenue Generated = Rs.110x = Rs. 352 crores
Therefore, Net Expenditure = Rs. 320 crores
and Profit = Rs. 32 crores

D Similarly, we can find Net Expenditure and Profit for other months as
B C well. The data has been tabulated below.

3 0°
Expenditure
Net
A’ Month on Raw
Expenditure
Material Profit
We have, 30° < ∠DAC + ∠DA 'C < 120°
(As ∠DA 'C = 30° and 0°< ∠DAC < 90°) January 320 83.2 32
Hence, the sum of the measures of ∠DAC + ∠DA 'C cannot February 400 128 72
be equal to 120° according to all the three cases. March 540 97.2 27
April 475 76 38
84. E Let the number of bananas with Moti, Sumit and Manky is a, b
and c, in no particular order. May 325 78 78
Then, a + b + c = 10 ...(i) June 550 209 77
1 1 1
Also, + + =1 ...(ii) All the values given above are in Rs. crores.
a b c

1 1 85. E The absolute percentage change in Net Expenditure over the


I. a = 1, then b + c = 9, + =0 previous month is highest in June and is equal to
b c
⇒ Not possible 550 − 325
× 100 ≈ 69%.
325
1 1 1
II. a = 2, then b + c = 8, + = ⇒ bc = 16
b c 2
86. C The maximum value of O-Expenditure was in the month of
⇒b=c=4 March and was equal to
1 1 2 21 82
III. a = 3, then b + c = 7, + = ⇒ bc = × 540 = Rs. 442.8 crores.
b c 3 2 100
⇒ Not possible
87. A Average expenditure on Raw Material
1 1 3
IV. a = 4, then b + c = 6, + = ⇒ bc = 8 671.4
b c 4 = = Rs. 111.9 crores
6
⇒ b = 2, c = 4 or b = 4, c = 2
1 1 4 88. C The increase in Profit over the previous month was more than
V. a = 5, then b + c = 5, + =
b c 5 100% in 2 months – February and May.

25
⇒ bc = ⇒ Not possible
4 89. C Sum of squares of first ‘n’ natural numbers is
n(n + 1)(2n + 1)
1 1 5
+ = ⇒ bc =
24 = A.
VI. a = 6, then b + c = 4, 6
b c 6 5
Now ‘n’ can take 7 types of values i.e.
⇒ Not possible
7k – 6, 7k – 5, 7k – 4, 7k – 3, 7k – 2, 7k – 1, and 7k, where k is
a natural number.
VII. a = 7, then b + c = 3
Thus, b and c must be 1 and 2, no particular order. (7k − 6)(7k − 5)(14k − 11)
From case I, none of b or c can be 1, hence not possible. If n = 7k – 6, A =
6
Hence, A will not be divisible by 7.
VIII. a = 8, then b + c = 2
Thus, each of b and c must be 1. (7k − 5)(7k − 4)(14k − 9)
From case I, none of b or c can be 1, hence not possible. If n = 7k – 5, A =
6
Hence, A will not be divisible by 7.
Thus, only possible solution for (a, b, c) is (2, 4, 4).
(7k − 4)(7k − 3)(14k − 7)
If n = 7k – 4, A =
6
Hence, A will be divisible by 7.
(7k − 3)(7k − 2)(14k − 5)
If n = 7k – 3, A =
6

MOCK XAT Page 11


Hence, A will not be divisible by 7. 150 – A ≥ 0 or 150 ≥ A
(7k − 2)(7k − 1)(14k − 3) 2A – 150 ≥ 0 or A ≥ 75
If n = 7k – 2, A =
6 300 – 3A ≥ 0 or 100 ≥ A
Hence, A will not be divisible by 7. 5A – 450 ≥ 0 or A ≥ 90
750 – 8A ≥ 0 or 93.75 ≥ A
(7k − 1)7k(14k − 1)
If n = 7k – 1, A = 13A – 1200 ≥ 0 or A ≥ 92.30
6
1950 – 21A ≥ 0 or 92.85 ≥ A
Hence, A will be divisible by 7.
7k(7k + 1)(14k + 1) So by taking A = 93, we get the first eight terms as non-
If n = 7k, A = negative
6
Hence, A will be divisible by 7.
94. C If two cars are moving in the opposite direction they take 35 s
Therefore, all the numbers of the type 7k – 4, 7k – 1 and 7k i.e. to meet.
3 numbers out of every 7 consecutive numbers will satisfy If the cars are moving in the same direction they take 210 s to
the given condition. So out of first 99 natural numbers 42 will meet.
satisfy the condition. Total such numbers = 42. As 210 is 6 times 35, this means that in 35 s, the faster car
1
90. B The given equation is ab + 2 = 2a + b + 600. must have traveled a distance equal to th of the circle more
6
⇒ ab – 2a – b + 2 = 600 ⇒ (a – 1)(b – 2) = 600
Since, it is given that ‘a’ is an odd number and ‘b’ is an even than that covered by the slower car.
number, which implies that both (a – 1) and (b – 2) are even If the slower car in 35 s travels a fraction of the circle equal to
numbers.  1 5
Therefore, the possible pairs of values of f, then f +  f +  = 1. Therefore, f = . The faster car
 6 12
(a – 1) and (b – 2) that satisfy the given equation are
(2, 300); (4, 150); (6, 100); (10, 60); (12, 50); (20, 30); travels
(30, 20); (50, 12); (60, 10); (100, 6); (150, 4); and (300, 2).  5 1 7
Therefore, there are 12 solutions for the given equation.  +  = th of the circle in 35s and hence it would take
 12 6  12

91. A As per the question: 60s to complete one round.


g(2, 2) = 2
g(2 + 1, 2) = g(3, 2) = g(2, 2) + 2 × 2 = 2 + 2 × 2 = 6 Alternate Method:
g(3, 2 + 1) = g(3, 3) = g(3, 2) – 2 × 2 = 6 – 2 × 2 = 2 Let R (in meters) be the circumference of the circle.
g(3 + 1, 3) = g(4, 3) = g(3, 3) + 2 × 3 = 2 + 2 × 3 = 8 Let the speeds (in m/s) of the faster and the slower car be ‘v’
and ‘u’ respectively.
Similarly: 35v + 35u = R and 210v – 210u = R
g(5, 4) = 2 × 4 + 2 = 10 Solving the above equations, we get that 60v = R.
g(6, 5) = 2 × 5 + 2 = 12 So, the faster car will take 60 seconds to complete one round.
Hence : g(1007, 1006) = 2 × 1006 + 2 = 2014
For questions 95 and 96:
92. B As a2 – 1 have to be positive, a2 – 1 > 0 or a2 > 1.
Also, a ≠ negative as 2a + 1 cannot be negative. A
Therefore, we get a > 1.
6 0°
Also, the largest angle is always opposite to the largest side.
Here the largest side is a2 + a + 1 as a > 1. Using cosine rule 90°
we get: Z
(2a + 1)2 + (a2 – 1)2 – (a2 + a + 1)2 
cosα=  
2(2a + 1)(a2 – 1)
B
1
Solving we get, cosα = – ⇒ α = 120°.
2 X
Y
93. C As we know that each term of the sequence should be non-
Let XYZ be the triangular park, right angled at Y and AB be the
negative, as soon as we get a negative term, this means the
tower with base at B.
sequence terminated at the previous term.
As B is the mid point of XZ, B is equidistant from each of X, Y and
a3 = 150 – A
Z.
a4 = 2A – 150
Therefore, we can conclude that A is also equidistant from each of
a5 = 300 – 3A
X, Y and Z.
a6 = 5A – 450
a7 = 750 – 8A
Let the length of edge YZ be 'x' meters.
a8 = 13A – 1200
a9 = 1950 – 21A
a10 = 34A – 3150

So to get the maximum number of terms all the terms should be


non-negative.

Page 12 MOCK XAT


I: Case II:
Consider p = 1 ⇒ (x − 3)(y − 1)(z − 1) = 1
A
⇒ x = 2, y = 2, z = 2 or x = 4, y = 2, z = 2

6 0°
Case III:
Consider p = 2 ⇒ (x − 3)(y − 1)(z − 1) = 2
⇒ x = 1, y = 2, z = 2 or x = 5, y = 2, z = 2
x m eters x m eters or x = 2 / 4, y = 3, z = 2 or x = 2 / 4, y = 2, z = 3
2+6 8
Probability that p = 1 or p = 2 = =
216 216

6 0° 6 0° 91 + 8 99
∴ Probability that p ≤ 2 = = = 45.83%
Y x m eters Z 216 216

98. B Cost of the product at the end of the last quarter of the financial
II: year 2009-2010 = Rs.10 lakhs.
Cost of the product at the end of the first quarter of the
A financial year 2010-2011
= 4.1% of 10 lakhs + 10 lakhs = Rs. 10.41 lakhs.
9 0° x m ete rs
x m ete rs
Similarly, approximate cost of the product at the end of second,
third and fourth quarters of financial year 2010-2011 was
4 5° 4 5° 10.66, 10.94, 11.33 lakhs respectively.
Y x 2 M eters X
So the average quarterly increase in cost
11.33 − 10 1.33
Therefore, length of AY = AZ = x meters and ≈ ≈
4 4
length of XY = x 2 meters ≈ Rs. 0.3325 lakh ≈ Rs.33,250.
1
Area of park = × XY × YZ = 2592 2 m2 99. D
2
3.2
⇒ x = 72 m and XZ = 72 3 m 20,000 = × Cost at the end of the last quarter of 2010-11
100
Cost at the end of the last quarter of 2010-11 = Rs. 6,25,000
2 Let the increase in cost at the end of March 2011 be Rs. x.
Height of the tower AB = AX2 − 
XZ 
95. C  = 36 meters Therefore, the cost of the product at the end of
 2  December 2010 = Rs. 6,25,000 – x
1.60
x= × (6,25,000 − x )
100
YZ + ZB or x ≈ Rs. 9,843.
96. A Required time =
20 × 10−2

=
72 + 36 3
20 × 10 −2
(
= 3 2 + 3 minutes)

97. A In order to find the probability of profit/loss, we need to find


the probability of p ≤ 2.
If p ≤ 2 ⇒ p = 0 or p = 1 or p = 2

Case I:
Consider p = 0 ⇒ (x − 3)(y − 1)(z − 1) = 0
⇒ x = 3 or y = 1, or z = 1
Probability that x = 3 or y = 1 or z = 1
= 1 – Probability that x ≠ 3, y ≠ 1 and z ≠ 1

5 5 5 91
= 1− × × =
6 6 6 216

MOCK XAT Page 13


100. C 101. E Using statement I:
Value of ‘a’ can be 127, 131, 137 or 139.
Apart from this nothing is known about ‘p’.
A Hence, statement I alone is not sufficient to answer the
question.

Using statement II:


By this statement if ‘p’ is even required remainder is 1 and if ‘p’
is odd, required remainder is 3. .
So, statement II alone is also not sufficient to answer the
question.

Using statement I and II together:


‘a’ will be 127, 131 or 139, when ‘p’ is odd and ‘a’ will be 137,
when ‘p’ is even.
30 ° B 60°
Therefore, statements I and II together are also not sufficient
P2 to answer the question. Hence, option E.
P1

Let AB be the pole with base at B.


Q P1 and P2 are two points on the ground,
∠ABP1 = ∠ABP2 = 90°

∠AP1 B = 30°

∴ BP1 = 3 h meters and ∠AP2B = 60°

h
∴ BP2 = meters
3
Now in ∆BP1P2 ,

h
(i) 3 h + > 20
3

4h
⇒ > 20
3

⇒h>5 3
h
(ii) 3 h − < 20
3

2h
⇒ < 20
3

⇒ h < 10 3
Combining (i) and (ii), we get 5 3 < h < 10 3.

Page 14 MOCK XAT

Вам также может понравиться